Proving the Integrability of a Given Function f on Interval [0,1]

  • Thread starter roam
  • Start date
  • Tags
    Functions
It's not rigorous, but it makes sense. In summary, the conversation revolved around the concept of integrability and how to prove that a given function is integrable. The participants discussed various definitions of integrability, including the use of Riemann sums and the relationship between antiderivatives and integrals. The conversation ended with a suggestion to argue that the area under the curve of the given function is 2, despite the function not being continuous at one point.
  • #1
roam
1,271
12
I need some help with this question please. Thanks. ::biggrin:

a) Let f: [0,1] [tex]\rightarrow R[/tex] be given by


http://img148.imageshack.us/img148/5435/10187431tc4.gif


Argue that f is integerable.




The attempt at a solution

I don't get it. A function is integerable if its integral exists. How can we integrate this? There is no actual function given here...

So how are we supposed to argue that it is integerable?

 
Last edited by a moderator:
Physics news on Phys.org
  • #2
What is the derivative of the following function?
[tex]g(x)= \left\{ \begin{array}{rl} 2x & x<1 \\ 3x & x=1 \end{array} [/tex]

edit- on the interval [0,1]
 
Last edited:
  • #3
gabbagabbahey said:
What is the derivative of the following function?
[tex]g(x)= \left\{ \begin{array}{rl} 2x & x<1 \\ 3x & x=1 \end{array} [/tex]

edit- on the interval [0,1]

I don't think this problem is about antiderivatives. It's about the definition of the integral in terms of Riemann sums. roam, it is a function. It just doesn't happen to be continuous. That doesn't mean it's not integrable. What's the area under the curve?
 
  • #4
Hi dick!

What's the area under the curve?

Uh, isn't it [tex]\int_{0}^{1}g(x) dx[/tex]?
 
  • #5
Hi! Sure it is, but what's the value of that? Think about Riemann upper sums and lower sums. You have to deal with the definition of the integral. If f(x)=2 the area is 2. Does moving only the single point at x=1 to f(1)=3 change that?
 
Last edited:
  • #6
Dick said:
I don't think this problem is about antiderivatives. It's about the definition of the integral in terms of Riemann sums. roam, it is a function. It just doesn't happen to be continuous. That doesn't mean it's not integrable. What's the area under the curve?

My Idea was to simply show that on the interval [0,1],
[tex]\frac{dg}{dx}=f(x)[/tex]
[tex]\Rightarrow \int_a^b f(x) dx= \int_a^b \frac{dg}{dx} dx =g(b)-g(a)[/tex]
for [tex](a,b) \epsilon [0,1][/tex] showing that the integral clearly exists and hence that f is integrable.
 
  • #7
But g(x) isn't differentiable at x=1, it's not even continuous. It doesn't make sense to say dg/dx=f at x=1. There are more basic definitions of an integral existing than that. Besides g(1)-g(0)=3. That's just plain silly. The area under the curve f(x) between 0 and 1 is 2.
 
  • #8
Dick said:
But g(x) isn't differentiable at x=1, it's not even continuous. It doesn't make sense to say dg/dx=f at x=1. There are more basic definitions of an integral existing than that. Besides g(1)-g(0)=3. That's just plain silly. The area under the curve f(x) between 0 and 1 is 2.

Good point, my mistake.
 
  • #9
Yes, functions that are not continuous can be integrable.
But Dick I'm not quite sure, we haven't studied some of that yet.

Btw, which definitions do I need to use for proving that f is integrable?

[tex]\sum_{k=0}^{1}k[/tex] [tex]\int_{0}^{1}f(x)dx = F(1)-F(0)[/tex] ?

Roam
 
  • #10
You haven't studied Riemann sums yet? Qualitatively, the idea is just that over ALMOST ALL of the interval the function is just f(x)=2. The bit where it jumps up to 3 only makes an 'infinitely small' (whatever that means) contribution to the area. If you don't have a rigorous definition of an integral (not antiderivatives - that's a theorem, not a definition), then you'll have to wave your hands and say stuff like that.
 
  • #11
This function has two values i.e.,
f(x) = 2 for all values of x below 1, so this value is integrable

The second value is
f(x) = 3 for x equal to 1, as there is only one number and not a range therefore this can not be integrated and it will have only one value.
thus the original function is integrable.

Does that make sense?
 
  • #12
I would recommend partitioning [0, 1] into n intervals with the last interval starting at 1- 1/n, ending of course at x= 1 so it has base length 1-(1- 1/n)= 1/n. You can take the maximum height of that interval to be 3 (the value of f at the right end point) so the maximum area is 3(1/n) which goes to 0 as n goes to infinity. The minimum height would be 2, the function value at the right end point (or anywhere else in the interval except x= 1) so the minimum area is 3(1/n) which also goes to 0 as n goes to infinity. Any other value for the height must lie between 2 and 3 and the area also goes to 0. That is sufficient to show that "f(x)= 3 for x= 1" contributes nothing to the integral. Use whatever method you like to show that f(x)= 2, for x between 0 and 2, is integrable.
 
  • #13
roam said:
This function has two values i.e.,
f(x) = 2 for all values of x below 1, so this value is integrable

The second value is
f(x) = 3 for x equal to 1, as there is only one number and not a range therefore this can not be integrated and it will have only one value.
thus the original function is integrable.

Does that make sense?

Sort of. You can integrate f(x)=3 from x=1 to x=1. The result is zero. What I really wanted was a DEFINITION of what an integral is. Nothing in the book or notes??
 
Last edited:
  • #14
Dick said:
What I really wanted was a DEFINITION of what an integral is. Nothing in the book or notes??

F is called an integral of f on interval I if F'(x) = f(x) for all x[tex]\in[/tex]I
 
  • #15
roam said:
F is called an integral of f on interval I if F'(x) = f(x) for all x[tex]\in[/tex]I

Then that leaves you in a pretty tough spot. The integrals F(x) of f(x) on the interval [0,1) (not including 1) are 2x+C for any constant. If you define F(1)=2+C then F'(1)=2 not 3. If you define it to be anything else then F doesn't have a derivative since it's not even continuous. Frustrating, huh? I think you are just stuck with arguing the area under the curve should be 2.
 

FAQ: Proving the Integrability of a Given Function f on Interval [0,1]

What is an Integerable Function?

An Integerable Function is a mathematical function that can be integrated using the techniques of calculus. This means that the function can be expressed as the limit of a sum of infinitely many small quantities, and the area under the curve of the function can be calculated.

What is the difference between an Integerable Function and a Non-Integerable Function?

The main difference between these two types of functions is that an Integerable Function can be integrated using calculus techniques, while a Non-Integerable Function cannot. This means that the area under the curve of a Non-Integerable Function cannot be calculated using traditional methods.

How do you determine if a function is Integerable?

A function is considered Integerable if it meets a set of specific criteria, such as being continuous and bounded on a given interval. Other factors, such as the type of discontinuities and the existence of antiderivatives, may also play a role in determining if a function is Integerable.

What are the benefits of working with Integerable Functions?

Integerable Functions have many practical applications, such as in physics, engineering, and economics. They allow for the calculation of important quantities, such as displacement, velocity, and acceleration, and can provide insights into the behavior of systems.

Can any function be made Integerable?

No, not all functions can be made Integerable. Some functions, such as those with infinite discontinuities or those that oscillate infinitely, cannot be integrated using traditional methods. However, there are techniques, such as the Riemann integral, that can be used to integrate a wider range of functions.

Back
Top